Showing the sum of this telescoping series

In summary, the divergence test is inconclusive, so I wrote as partial fractions and started analysing the nth sum.
  • #1
Euler2718
90
3

Homework Statement



Determine whether each of the following series is convergent or divergent. If the series is convergent, find its sum

[tex] \sum_{i=1}^{\infty} \frac{6}{9i^{2}+6i-8} [/tex]

Homework Equations



Partial fraction decomposition

[tex] \frac{1}{3i-2} - \frac{1}{3i+4} [/tex]

The Attempt at a Solution



The divergence test is inconclusive, so I wrote as partial fractions and started analysing the nth sum:

[tex] S_{n} = \left( 1-\frac{1}{7} \right) + \left( \frac{1}{4} - \frac{1}{10} \right) + \left( \frac{1}{7} - \frac{1}{13} \right) + \left( \frac{1}{10} - \frac{1}{16} \right) + \dots [/tex]

1 and 1/4 are the only terms that do not cancel, but how do I show this in the nth case? I'm having trouble writing it generally.
 
Physics news on Phys.org
  • #2
Morgan Chafe said:

Homework Statement



Determine whether each of the following series is convergent or divergent. If the series is convergent, find its sum

[tex] \sum_{i=1}^{\infty} \frac{6}{9i^{2}+6i-8} [/tex]

Homework Equations



Partial fraction decomposition

[tex] \frac{1}{3i-2} - \frac{1}{3i+4} [/tex]

The Attempt at a Solution



The divergence test is inconclusive, so I wrote as partial fractions and started analysing the nth sum:

[tex] S_{n} = \left( 1-\frac{1}{7} \right) + \left( \frac{1}{4} - \frac{1}{10} \right) + \left( \frac{1}{7} - \frac{1}{13} \right) + \left( \frac{1}{10} - \frac{1}{16} \right) + \dots [/tex]

1 and 1/4 are the only terms that do not cancel, but how do I show this in the nth case? I'm having trouble writing it generally.
Include the general term in your expansion:
##S_{n} = \left( 1-\frac{1}{7} \right) + \left( \frac{1}{4} - \frac{1}{10} \right) + \left( \frac{1}{7} - \frac{1}{13} \right) + \left( \frac{1}{10} - \frac{1}{16} \right) + \dots + \left( \frac{1}{3n-2} - \frac{1}{3n+4} \right) + \dots##
If you add in the term before and the one after the last term I wrote above, you should see how the telescoping happens.
 
  • Like
Likes Euler2718
  • #3
Morgan Chafe said:

Homework Statement



Determine whether each of the following series is convergent or divergent. If the series is convergent, find its sum

[tex] \sum_{i=1}^{\infty} \frac{6}{9i^{2}+6i-8} [/tex]

Homework Equations



Partial fraction decomposition

[tex] \frac{1}{3i-2} - \frac{1}{3i+4} [/tex]

The Attempt at a Solution



The divergence test is inconclusive, so I wrote as partial fractions and started analysing the nth sum:

[tex] S_{n} = \left( 1-\frac{1}{7} \right) + \left( \frac{1}{4} - \frac{1}{10} \right) + \left( \frac{1}{7} - \frac{1}{13} \right) + \left( \frac{1}{10} - \frac{1}{16} \right) + \dots [/tex]

1 and 1/4 are the only terms that do not cancel, but how do I show this in the nth case? I'm having trouble writing it generally.

Show that for every term of the form ##1/(2n)## there will be another term ##-1/(2n)## (corresponding to just two possible values of ##i##) and for every term ##1/(2n+1)## there is a cancelling term ##-1/(2n+1)##---again, corresponding to exactly two values of ##i##.
 
  • Like
Likes Euler2718
  • #4
Ray Vickson said:
Show that for every term of the form ##1/(2n)## there will be another term ##-1/(2n)## (corresponding to just two possible values of ##i##) and for every term ##1/(2n+1)## there is a cancelling term ##-1/(2n+1)##---again, corresponding to exactly two values of ##i##.
Mark44 said:
Include the general term in your expansion:
##S_{n} = \left( 1-\frac{1}{7} \right) + \left( \frac{1}{4} - \frac{1}{10} \right) + \left( \frac{1}{7} - \frac{1}{13} \right) + \left( \frac{1}{10} - \frac{1}{16} \right) + \dots + \left( \frac{1}{3n-2} - \frac{1}{3n+4} \right) + \dots##
If you add in the term before and the one after the last term I wrote above, you should see how the telescoping happens.

Thanks guys, I got it now.
 

Related to Showing the sum of this telescoping series

1. What is a telescoping series?

A telescoping series is a mathematical series where most of the terms cancel each other out, leaving only a finite number of terms to be added together.

2. How do you determine the sum of a telescoping series?

To determine the sum of a telescoping series, you can use the method of partial fractions or the method of finite differences. Both methods involve breaking down the series into simpler terms that can be more easily summed.

3. What is the general formula for calculating the sum of a telescoping series?

The general formula for calculating the sum of a telescoping series is S = lim n→∞ (sn), where S is the sum of the series and sn is the nth partial sum of the series.

4. Can you provide an example of a telescoping series?

One example of a telescoping series is the series 1 + 1/2 + 1/4 + 1/8 + ... = 2. In this series, most of the terms cancel each other out, leaving only the first term and the last term, resulting in a sum of 2.

5. How can telescoping series be applied in real-world situations?

Telescoping series can be applied in various real-world situations, such as in calculating the sum of infinite geometric series, in evaluating limits, and in solving differential equations. They can also be used in finance to calculate the present value of an annuity or in physics to calculate the position of an object in motion.

Similar threads

  • Calculus and Beyond Homework Help
Replies
1
Views
306
  • Calculus and Beyond Homework Help
Replies
1
Views
261
  • Calculus and Beyond Homework Help
Replies
2
Views
237
  • Calculus and Beyond Homework Help
Replies
3
Views
440
  • Calculus and Beyond Homework Help
Replies
5
Views
921
  • Calculus and Beyond Homework Help
Replies
2
Views
729
  • Calculus and Beyond Homework Help
Replies
2
Views
752
  • Calculus and Beyond Homework Help
Replies
7
Views
1K
  • Calculus and Beyond Homework Help
Replies
17
Views
1K
  • Calculus and Beyond Homework Help
Replies
29
Views
1K
Back
Top